\documentclass[11pt]{article} \usepackage{personal_commands} \usepackage[italian]{babel} \title{\textbf{Note del corso di Geometria 1}} \author{Gabriel Antonio Videtta} \date{27 e 31 marzo 2023} \begin{document} \maketitle \begin{center} \Large \textbf{Proprietà e teoremi principali sul prodotto scalare} \end{center} \begin{note} Nel corso del documento, per $V$ si intenderà uno spazio vettoriale di dimensione finita $n$ e per $\varphi$ un suo prodotto scalare. Analogamente si intenderà lo stesso per $V'$ e $\varphi'$. \end{note} \begin{proposition}[formula delle dimensioni del prodotto scalare] Sia $W \subseteq V$ un sottospazio di $V$. Allora vale la seguente identità: \[ \dim W + \dim W^\perp = \dim V + \dim (W \cap V^\perp). \] \end{proposition} \begin{proof} Si consideri l'applicazione lineare $a_\varphi$ introdotta precedentemente. Si osserva che $W^\perp = \Ker (i^\top \circ a_\varphi)$, dove $i : W \to V$ è tale che $i(\vec w) = \vec w$. Allora, per la formula delle dimensioni, vale la seguente identità: \begin{equation} \label{eq:dim_formula_dimensioni_1} \dim V = \dim W^\perp + \rg (i^\top \circ a_\varphi). \end{equation} \vskip 0.05in Sia allora $f = i^\top \circ a_\varphi$. Si consideri ora l'applicazione $g = a_\varphi \circ i : W \to \dual V$. Sia ora $\basis_W$ una base di $W$ e $\basis_V$ una base di $V$. Allora le matrici associate di $f$ e di $g$ sono le seguenti: \begin{enumerate}[(i)] \item $M_{\dual \basis_W}^{\basis_V}(f) = M_{\dual \basis_W}^{\basis_V}(i^\top \circ a_\varphi) = \underbrace{M_{\dual \basis_W}^{\dual \basis_V}(i^\top)}_A \underbrace{M_{\dual \basis_V}^{\basis_V}(a_\varphi)}_B = AB$, \item $M_{\dual \basis_V}^{\basis_W}(g) = M_{\dual \basis_V}^{\basis_W}(a_\varphi \circ i) = \underbrace{M_{\dual \basis_V}^{\basis_V}(a_\varphi)}_B \underbrace{M_{\basis_V}^{\basis_W}(i)}_{A^\top} = BA^\top \overbrace{=}^{B^\top = B} (AB)^\top$. \end{enumerate} Poiché $\rg(A) = \rg(A^\top)$, si deduce che $\rg(f) = \rg(g) \implies \rg(i^\top \circ a_\varphi) = \rg(a_\varphi \circ i) = \rg(\restr{a_\varphi}{W}) = \dim W - \dim \Ker \restr{a_\varphi}{W}$, ossia che: \begin{equation} \label{eq:dim_formula_dimensioni_2} \rg(i^\top \circ a_\varphi) = \dim W - \dim (W \cap \underbrace{\Ker a_\varphi}_{V^\perp}) = \dim W - \dim (W \cap V^\perp). \end{equation} Si conclude allora, sostituendo l'equazione \eqref{eq:dim_formula_dimensioni_2} nell'equazione \eqref{eq:dim_formula_dimensioni_1}, che $\dim V = \dim W^\top + \dim W - \dim (W \cap V^\perp)$, ossia la tesi. \end{proof} \begin{remark} Si identifica $\w^\perp$ come il sottospazio di tutti i vettori di $V$ ortogonali a $\w$. In particolare, se $W = \Span(\vec w)$ è il sottospazio generato da $\vec w \neq \vec 0$, $\vec w \in V$, allora $W^\perp = \w^\perp$. Inoltre valgono le seguenti equivalenze: $\vec w \notin W^\perp \iff$ $\Rad (\restr{\varphi}{W}) = W \cap W^\perp = \zerovecset$ $\iff \vec w \text{ non è isotropo } \iff$ $V = W \oplus W^\perp$. \end{remark} \begin{definition} Si definisce \textbf{base ortogonale} di $V$ una base $\vv 1$, ..., $\vv n$ tale per cui $\varphi(\vv i, \vv j) = 0 \impliedby i \neq j$, ossia una base per cui la matrice associata del prodotto scalare è diagonale. \end{definition} \begin{proposition}[formula di polarizzazione] Se $\Char \KK \neq 2$, un prodotto scalare è univocamente determinato dalla sua forma quadratica $q$. In particolare vale la seguente identità: \[ \varphi(\v, \w) = \frac{q(\v + \w) - q(\v) - q(\w)}{2}. \] \vskip 0.05in \end{proposition} \begin{proof} Si osserva che $q(\vec v + \vec w) - q(\vec v) - q(\vec w) = 2 \varphi(\vec v, \vec w)$, e quindi, poiché $2$ è invertibile per ipotesi, si deduce che $\varphi(\vec v, \vec w) = \frac{q(\vec v + \vec w) - q(\vec v) - q(\vec w)}{2}$. \end{proof} \begin{theorem}[di Lagrange] Ogni spazio vettoriale $V$ su $\KK$ tale per cui $\Char \KK \neq 2$ ammette una base ortogonale. \end{theorem} \begin{proof} Si dimostra il teorema per induzione su $n := \dim V$. Per $n \leq 1$, la tesi è triviale (ogni base è già una base ortogonale). Sia allora il teorema vero per $i \leq n$. Se $V$ ammette un vettore non isotropo $\vec w$, sia $W = \Span(\vec w)$, e si consideri la decomposizione $V = W \oplus W^\perp$. Poiché $W^\perp$ ha dimensione $n-1$, per ipotesi induttiva ammette una base ortogonale. Inoltre, tale base è anche ortogonale a $W$, e quindi l'aggiunta di $\vec w$ a questa base ne fa una base ortogonale di $V$. Se invece $V$ non ammette vettori non isotropi, ogni forma quadratica è nulla, e quindi il prodotto scalare è nullo per la proposizione precedente. Allora in questo caso ogni base è una base ortogonale, completando il passo induttivo, e dunque la dimostrazione. \end{proof} \begin{note} D'ora in poi, nel corso del documento, si assumerà $\Char \KK \neq 2$. \end{note} \begin{theorem}[di Sylvester, caso complesso] Sia $\KK$ un campo i cui elementi sono tutti quadrati di un altro elemento del campo (e.g.~$\CC$). Allora esiste una base ortogonale $\basis$ tale per cui: \[ M_\basis(\varphi) = \Matrix{I_r & \rvline & 0 \\ \hline 0 & \rvline & 0\,}. \] \end{theorem} \begin{proof} Per il teorema di Lagrange, esiste una base ortogonale $\basis'$ di $V$. Si riordini allora la base $\basis'$ in modo tale che la forma quadratica valutata nei primi elementi sia sempre diversa da zero. Allora, poiché ogni elemento di $\KK$ è per ipotesi quadrato di un altro elemento di $\KK$, si sostituisca $\basis'$ con una base $\basis$ tale per cui, se $q(\vv i) = 0$, $\vv i \mapsto \vv i$, e altrimenti $\vv i \mapsto \frac{\vv i}{\sqrt{q(\vv i)}}$. Allora $\basis$ è una base tale per cui la matrice associata del prodotto scalare in tale base è proprio come desiderata nella tesi, dove $r$ è il numero di elementi tali per cui la forma quadratica valutata in essi sia diversa da zero. \end{proof} \begin{remark} Si possono effettuare alcune considerazioni sul teorema di Sylvester complesso. \\ \li Si può immediatamente concludere che il rango è un invariante completo per la congruenza in un campo in cui tutti gli elementi sono quadrati, ossia che $A \cong B \iff \rg(A) = \rg(B)$, se $A$ e $B$ sono matrici simmetriche: infatti ogni matrice simmetrica rappresenta una prodotto scalare, ed è pertanto congruente ad una matrice della forma desiderata nell'enunciato del teorema di Sylvester complesso. Poiché il rango è un invariante della congruenza, si ricava che $r$ nella forma della matrice di Sylvester, rappresentando il rango, è anche il rango di ogni sua matrice congruente. In particolare, se due matrici simmetriche hanno lo stesso rango, allora sono congruenti alla stessa matrice di Sylvester, e quindi, essendo la congruenza una relazione di equivalenza, sono congruenti a loro volta tra di loro. \\ \li Due matrici simmetriche con stesso rango, allora, non solo sono SD-equivalenti, ma sono anche congruenti. \\ \li Ogni base ortogonale deve quindi avere lo stesso numero di elementi nulli. \end{remark} \begin{definition}[somma diretta ortogonale] Siano i sottospazi $U$ e $W \subseteq V$ in somma diretta. Allora si dice che $U$ e $W$ sono in \textbf{somma diretta ortogonale} rispetto al prodotto scalare $\varphi$ di $V$, ossia che $U \oplus W = U \oplus^\perp W$, se $\varphi(\vec u, \vec w) = 0$ $\forall \vec u \in U$, $\vec w \in W$. \end{definition} \begin{definition}[cono isotropo] Si definisce \textbf{cono isotropo} di $V$ rispetto al prodotto scalare $\varphi$ il seguente insieme: \[ \CI(\varphi) = \{ \v \in V \mid \varphi(\v, \v) = 0 \}, \] \vskip 0.05in ossia l'insieme dei vettori isotropi di $V$. \end{definition} \begin{note} La notazione $\varphi > 0$ indica che $\varphi$ è definito positivo (si scrive $\varphi \geq 0$ se invece è semidefinito positivo). Analogamente $\varphi < 0$ indica che $\varphi$ è definito negativo (e $\varphi \leq 0$ indica che è semidefinito negativo). \end{note} \begin{exercise} Sia $\Char \KK \neq 2$. Siano $\vv1$, ..., $\vv k \in V$ e sia $M = \left( \varphi(\vv i, \vv j) \right)_{i, j = 1\textrm{---}k} \in M(k, \KK)$, dove $\varphi$ è un prodotto scalare di $V$. Sia inoltre $W = \Span(\vv 1, ..., \vv k)$. Si dimostrino allora le seguenti affermazioni. \begin{enumerate}[(i)] \item Se $M$ è invertibile, allora $\vv 1$, ..., $\vv k$ sono linearmente indipendenti. \item Siano $\vv 1$, ..., $\vv k$ linearmente indipendenti. Allora $M$ è invertibile $\iff$ $\restr{\varphi}{W}$ è non degenere $\iff$ $W \cap W^\perp = \zerovecset$. \item Siano $\vv1$, ..., $\vv k$ a due a due ortogonali tra loro. Allora $M$ è invertibile $\iff$ nessun vettore $\vv i$ è isotropo. \item Siano $\vv1$, ..., $\vv k$ a due a due ortogonali tra loro e siano anche linearmente indipendenti. Allora $M$ è invertibile $\implies$ si può estendere $\basis_W = \{\vv 1, \ldots, \vv k\}$ a una base ortogonale di $V$. \item Sia $\KK = \RR$. Sia inoltre $\varphi > 0$. Allora $\vv 1$, ..., $\vv k$ sono linearmente indipendenti $\iff$ $M$ è invertibile. \item Sia $\KK = \RR$. Sia ancora $\varphi > 0$. Allora se $\vv 1$, ..., $\vv k$ sono a due a due ortogonali e sono tutti non nulli, sono anche linearmente indipendenti. \end{enumerate} \end{exercise} \begin{solution} \begin{enumerate}[(i)] \item Siano $a_1$, ..., $a_k \in \KK$ tali che $a_1 \vv 1 + \ldots + a_k \vv k = 0$. Vale in particolare che $\vec 0 = \varphi(\vv i, \vec 0) = \varphi(\vv i, a_1 \vv 1 + \ldots + a_k \vv k) = \sum_{j=1}^k a_j \varphi(\vv i, \vv j)$ $\forall 1 \leq i \leq k$. Allora $\sum_{j=1}^k a_j M^j = 0$. Dal momento che $M$ è invertibile, $\rg(M) = k$, e quindi l'insieme delle colonne di $M$ è linearmente indipendente, da cui si ricava che $a_j = 0$ $\forall 1 \leq j \leq k$, e quindi che $\vv 1$, ..., $\vv k$ sono linearmente indipendenti. \item Poiché $\vv 1$, ..., $\vv k$ sono linearmente indipendenti, tali vettori formano una base di $W$, detta $\basis$. In particolare, allora, vale che $M = M_\basis(\restr{\varphi}{W})$. Pertanto, se $M$ è invertibile, $\Rad(\restr{\varphi}{W}) = \Ker M = \zerovecset$, e dunque $\restr{\varphi}{W}$ è non degenere. Se invece $\restr{\varphi}{W}$ è non degenere, $\zerovecset = \Rad(\restr{\varphi}{W}) = W \cap W^\perp$. Infine, se $W \cap W^\perp = \zerovecset$, $\zerovecset = W \cap W^\perp = \Rad(\restr{\varphi}{W}) = \Ker M$, e quindi $M$ è iniettiva, e dunque invertibile. \item Dal momento che $\vv 1$, ..., $\vv k$ sono ortogonali tra loro, $M$ è una matrice diagonale. Pertanto $M$ è invertibile se e solo se ogni suo elemento diagonale è diverso da $0$, ossia se $\varphi(\vv i, \vv i) \neq 0$ $\forall 1 \leq i \leq k$, e dunque se e solo se nessun vettore $\vv i$ è isotropo. \item Se $M$ è invertibile, da (ii) si deduce che $\Rad(\restr{\varphi}{W}) = W \cap W^\perp = \zerovecset$, e quindi che $W$ e $W^\perp$ sono in somma diretta. Inoltre, per la formula delle dimensioni del prodotto scalare, $\dim W + \dim W^\perp = \dim V + \underbrace{\dim (W \cap V^\perp)}_{\leq \dim (W \cap W^\perp) = 0} = \dim V$. Pertanto $V = W \oplus^\perp W^\perp$. \\ Allora, dacché $\Char \KK \neq 2$, per il teorema di Lagrange, $W^\perp$ ammette una base ortogonale $\basis_{W^\perp}$. Si conclude dunque che $\basis = \basis_W \cup \basis_{W^\perp}$ è una base ortogonale di $V$. \item Se $M$ è invertibile, da (i) $\vv1$, ..., $\vv k$ sono linearmente indipendenti. Siano ora invece $\vv 1$, ..., $\vv k$ linearmente indipendenti per ipotesi. Siano $a_1$, ..., $a_k \in \KK$ tali che $a_1 M^1 + \ldots + a_k M^k = 0$, allora $a_1 \varphi(\vv i, \vv 1) + \ldots + a_k \varphi(\vv i, \vv k) = 0$ $\forall 1 \leq i \leq k$. Pertanto, detto $\v = a_1 \vv 1 + \ldots + a_k \vv k$, si ricava che: \[ \varphi(\v, \v) = \sum_{i=1}^k \sum_{j=1}^k a_j \, \varphi(\vv i, \vv j) = 0. \] Tuttavia questo è possibile solo se $\v = a_1 \vv 1 + \ldots + a_k \vv k = 0$. Dal momento che $\vv 1$, ..., $\vv k$ sono linearmente indipendenti, si conclude che $a_1 = \cdots = a_k = 0$, ossia che le colonne di $M$ sono tutte linearmente indipendenti e quindi che $\rg(M) = k \implies$ $M$ è invertibile. \item Poiché $\vv 1$, ..., $\vv k$ sono ortogonali a due a due tra loro, $M$ è una matrice diagonale. Inoltre, dacché $\varphi > 0$ e $\vv i \neq \vec 0$ $\forall 1 \leq i \leq k$, gli elementi diagonali di $M$ sono sicuramente tutti diversi da zero, e quindi $\det (M) \neq 0$ $\implies$ $M$ è invertibile. Allora, per il punto (v), $\vv 1$, ..., $\vv k$ sono linearmente indipendenti. \end{enumerate} \end{solution} \begin{definition} Data una base ortogonale $\basis$ di $V$ rispetto al prodotto scalare $\varphi$, si definiscono i seguenti indici: \begin{align*} \iota_+(\varphi) &= \max\{ \dim W \mid W \subseteq V \E \restr{\varphi}{W} > 0 \}, &\text{(}\textbf{indice di positività}\text{)} \\ \iota_-(\varphi) &= \max\{ \dim W \mid W \subseteq V \E \restr{\varphi}{W} < 0 \}, &\text{(}\textbf{indice di negatività}\text{)}\\ \iota_0(\varphi) &= \dim V^\perp. &\text{(}\textbf{indice di nullità}\text{)} \end{align*} Quando il prodotto scalare $\varphi$ è noto dal contesto, si omette e si scrive solo $\iota_+$, $\iota_-$ e $\iota_0$. In particolare, la terna $\sigma(\varphi) = \sigma = (i_+, i_-, i_0)$ è detta \textbf{segnatura} del prodotto $\varphi$. \end{definition} \begin{theorem}[di Sylvester, caso reale] Sia $\KK$ un campo ordinato i cui elementi positivi sono tutti quadrati (e.g.~$\RR$). Allora esiste una base ortogonale $\basis$ tale per cui: \[ M_\basis(\varphi) = \Matrix{I_{\iota_+} & \rvline & 0 & \rvline & 0 \\ \hline 0 & \rvline & -I_{\iota_-} & \rvline & 0 \\ \hline 0 & \rvline & 0 & \rvline & 0\cdot I_{\iota_0} }. \] \vskip 0.05in Inoltre, per ogni base ortogonale, esistono esattamente $\iota_+$ vettori della base con forma quadratica positiva, $\iota_-$ con forma negativa e $\iota_0$ con forma nulla. \end{theorem} \begin{proof} Per il teorema di Lagrange, esiste una base ortogonale $\basis'$ di $V$. Si riordini la base in modo tale che la forma quadratica valutata nei primi elementi sia strettamente positiva, che nei secondi elementi sia strettamente negativa e che negli ultimi sia nulla. Si sostituisca $\basis'$ con una base $\basis$ tale per cui, se $q(\vv i) > 0$, allora $\vv i \mapsto \frac{\vv i}{\sqrt{q(\vv i)}}$; se $q(\vv i) < 0$, allora $\vv i \mapsto \frac{\vv i}{\sqrt{-q(\vv i)}}$; altrimenti $\vv i \mapsto \vv i$. Si è allora trovata una base la cui matrice associata del prodotto scalare è come desiderata nella tesi. \\ Sia ora $a$ il numero di vettori della base con forma quadratica positiva, $b$ il numero di vettori con forma negativa e $c$ quello dei vettori con forma nulla. Si consideri $W_+ = \Span(\vv 1, ..., \vv a)$, $W_- = \Span(\vv{a+1}, ..., \vv b)$, $W_0 = \Span(\vv{b+1}, ..., \vv c)$. \\ Sia $M = M_\basis(\varphi)$. Si osserva che $c = n - \rg(M) = \dim \Ker(M) = \dim V^\perp = \iota_0$. Inoltre $\forall \v \in W_+$, dacché $\basis$ è ortogonale, $q(\v) = q(\sum_{i=1}^a \alpha_i \vv i) = \sum_{i=1}^a \alpha_i^2 q(\vv i) > 0$, e quindi $\restr{\varphi}{W_+} > 0$, da cui $\iota_+ \geq a$. Analogamente $\iota_- \geq b$. \\ Si mostra ora che è impossibile che $\iota_+ > a$. Se così infatti fosse, sia $W$ tale che $\dim W = \iota_+$ e che $\restr{\varphi}{W} > 0$. $\iota_+ + b + c$ sarebbe maggiore di $a + b + c = n := \dim V$. Quindi, per la formula di Grassman, $\dim(W + W_- + W_0) = \dim W + \dim(W_- + W_0) - \dim (W \cap (W_- + W_0)) \implies \dim (W \cap (W_- + W_0)) = \dim W + \dim(W_- + W_0) - \dim(W + W_- + W_0) > 0$, ossia esisterebbe $\v \neq \{\vec 0\} \mid \v \in W \cap (W_- + W_0)$. Tuttavia questo è assurdo, dacché dovrebbe valere sia $q(\v) > 0$ che $q(\v) < 0$, \Lightning. Quindi $\iota_+ = a$, e analogamente $\iota_- = b$. \end{proof} \begin{definition} Si dice \textbf{base di Sylvester} una base di $V$ tale per cui la matrice associata di $\varphi$ sia esattamente nella forma vista nella dimostrazione del teorema di Sylvester. Analogamente si definisce tale matrice come \textbf{matrice di Sylvester}. \end{definition} \begin{remark} \nl \li Si può dunque definire la segnatura di una matrice simmetrica come la segnatura di una qualsiasi sua base ortogonale, dal momento che tale segnatura è invariante per cambiamento di base. \\ \li La segnatura è un invariante completo per la congruenza nel caso reale. Se infatti due matrici hanno la stessa segnatura, sono entrambe congruenti alla matrice come vista nella dimostrazione della forma reale del teorema di Sylvester, e quindi, essendo la congruenza una relazione di equivalenza, sono congruenti tra loro. Analogamente vale il viceversa, dal momento che ogni base ortogonale di due matrici congruenti devono contenere gli stessi numeri $\iota_+$, $\iota_-$ e $\iota_0$ di vettori di base con forma quadratica positiva, negativa e nulla. \\ \li Se $\ww 1$, ..., $\ww k$ sono tutti i vettori di una base ortogonale $\basis$ con forma quadratica nulla, si osserva che $W = \Span(\ww 1, ..., \ww k)$ altro non è che $V^\perp$ stesso. Infatti, come visto anche nella dimostrazione del teorema di Sylvester reale, vale che $\dim W = \dim \Ker (M_\basis(\varphi)) = \dim V^\perp$. Inoltre, se $\w \in W$ e $\v \in V$, $\varphi(\w, \v) = \varphi(\sum_{i=1}^k \alpha_i \ww i, \sum_{i=1}^k \beta_i \ww i + \sum_{i=k+1}^n \beta_i \vv i) = \sum_{i=1}^k \alpha_i \beta_i q(\ww i) = 0$, e quindi $W \subseteq V^\perp$, da cui si conclude che $W = V^\perp$. \\ \li Vale in particolare che $\rg(\varphi) = \iota_+ + \iota_-$, mentre $\dim \Ker(\varphi) = \iota_0$, e quindi $n = \iota_+ + \iota_- + \iota_0$. \\ \li Se $V = U \oplusperp W$, allora $\iota_+(\varphi) = \iota_+(\restr{\varphi}{V}) + \iota_+(\restr{\varphi}{W})$, e analogamente per gli altri indici. \end{remark} \begin{definition}[isometria tra due spazi vettoriali] Dati due spazi vettoriali $(V, \varphi)$ e $(V', \varphi')$ dotati di prodotto scalare sullo stesso campo $\KK$, si dice che $V$ e $V'$ sono \textbf{isometrici} se esiste un isomorfismo $f$, detto \textit{isometria}, che preserva tali che prodotti, ossia tale che: \[ \varphi(\vec v, \vec w) = \varphi'(f(\vec v), f(\vec w)). \] \end{definition} \begin{exercise} Sia $f : V \to V'$ un isomorfismo. Allora $f$ è un'isometria $\iff$ $\forall$ base $\basis = \{ \vv 1, \ldots, \vv n \}$ di $V$, $\basis' = \{ f(\vv 1), \ldots, f(\vv n) \}$ è una base di $V'$ e $\varphi(\vv i, \vv j) = \varphi'(f(\vv i), f(\vv j))$ $\forall 1 \leq i, j \leq n$ $\iff$ $\exists$ base $\basis = \{ \vv 1, \ldots, \vv n \}$ di $V$ tale che $\basis' = \{ f(\vv 1), \ldots, f(\vv n) \}$ è una base di $V'$ e $\varphi(\vv i, \vv j) = \varphi'(f(\vv i), f(\vv j))$ $\forall 1 \leq i, j \leq n$. \end{exercise} \begin{solution} Se $f$ è un'isometria, detta $\basis$ una base di $V$, $\basis' = f(\basis)$ è una base di $V'$ dal momento che $f$ è anche un isomorfismo. Inoltre, dacché $f$ è un'isometria, vale sicuramente che $\varphi(\vv i, \vv j) = \varphi'(f(\vv i), f(\vv j))$ $\forall 1 \leq i, j \leq n$. \\ Sia ora assunto per ipotesi che $\forall$ base $\basis = \{ \vv 1, \ldots, \vv n \}$ di $V$, $\basis' = \{ f(\vv 1), \ldots, f(\vv n) \}$ è una base di $V'$ e $\varphi(\vv i, \vv j) = \varphi'(f(\vv i), f(\vv j))$ $\forall 1 \leq i, j \leq n$. Allora, analogamente a prima, detta $\basis = \{ \vv 1, \ldots, \vv n \}$ una base di $V$, $\basis' = f(\basis)$ è una base di $V'$, e in quanto tale, per ipotesi, è tale che $\varphi(\vv i, \vv j) = \varphi'(f(\vv i), f(\vv j))$ $\forall 1 \leq i, j \leq n$. \\ Sia infine assunto per ipotesi che $\exists$ base $\basis = \{ \vv 1, \ldots, \vv n \}$ di $V$ tale che $\basis' = \{ f(\vv 1), \ldots, f(\vv n) \}$ è una base di $V'$ e $\varphi(\vv i, \vv j) = \varphi'(f(\vv i), f(\vv j))$ $\forall 1 \leq i, j \leq n$. Siano $\v$, $\w \in V$. Allora $\exists a_1$, ..., $a_n$, $b_1$, ..., $b_n \in \KK$ tali che $\v = a_1 \vv 1 + \ldots + a_n \vv n$ e $\w = b_1 \vv 1 + \ldots + b_n \vv n$. Si ricava pertanto che: \[ \varphi'(f(\v), f(\w)) = \sum_{i=1}^n \sum_{j=1}^n a_i b_j \, \varphi'(f(\vv i), f(\vv j)) = \sum_{i=1}^n \sum_{j=1}^n a_i b_j \, \varphi(\vv i, \vv j) = \varphi(\v, \w), \] da cui la tesi. \end{solution} \begin{proposition} Sono equivalenti le seguenti affermazioni: \begin{enumerate}[(i)] \item $V$ e $V'$ sono isometrici; \item $\forall$ base $\basis$ di $V$, base $\basis'$ di $V'$, $M_\basis(\varphi)$ e $M_{\basis'}(\varphi')$ sono congruenti; \item $\exists$ base $\basis$ di $V$, base $\basis'$ di $V'$ tale che $M_\basis(\varphi)$ e $M_{\basis'}(\varphi')$ sono congruenti. \end{enumerate} \end{proposition} \begin{proof} Se $V$ e $V'$ sono isometrici, sia $f : V \to V'$ un'isometria. Sia $\basisC = \{ \vv 1, \ldots, \vv n \}$ una base di $V$. Allora, poiché $f$ è anche un isomorfismo, $\basisC' = f(\basisC)$ è una base di $V$ tale che $\varphi(\vv i, \vv j) = \varphi'(f(\vv i), f(\vv j))$ $\forall 1 \leq i, j \leq n$. Pertanto $M_\basisC(\varphi) = M_{\basisC'}(\varphi')$. Si conclude allora che, cambiando base in $V$ (o in $V'$), la matrice associata al prodotto scalare varia per congruenza dalla formula di cambiamento di base per il prodotto scalare, da cui si ricava che per ogni scelta di $\basis$ base di $V$ e di $\basis'$ base di $V'$, $M_\basis(\varphi) \cong M_{\basis'}(\varphi')$. Inoltre, se tale risultato è vero per ogni $\basis$ base di $V$ e di $\basis'$ base di $V'$, dal momento che sicuramente esistono due basi $\basis$, $\basis'$ di $V$ e $V'$, vale anche (ii) $\implies$ (iii). \\ Si dimostra ora (iii) $\implies$ (i). Per ipotesi $M_\basis(\varphi) \cong M_{\basis'}(\varphi')$, quindi $\exists P \in \GL(n, \KK) \mid M_{\basis'}(\varphi') = P^\top M_\basis(\varphi) P$. Allora $\exists$ $\basis''$ base di $V'$ tale che $P = M_{\basis''}^{\basis'}(\Idv)$, da cui $P\inv = M_{\basis'}^{\basis''}(\varphi)$. Per la formula di cambiamento di base del prodotto scalare, $M_{\basis''}(\varphi) = (P\inv)^\top M_{\basis'} P\inv = M_\basis(\varphi)$. Detta $\basis'' = \{ \ww 1, \ldots, \ww n \}$, si costruisce allora l'isomorfismo $f : V \to V'$ tale che $f(\vv i) = \ww i$ $\forall 1 \leq i \leq n$.. Dal momento che per costruzione $M_\basis(\varphi) = M_{\basis''}(\varphi')$, $\varphi(\vv i, \vv j) = \varphi'(\ww i, \ww j) = \varphi'(f(\vv i), f(\vv j))$ $\forall 1 \leq i, j \leq n$. Si conclude dunque che $\varphi(\v, \w) = \varphi'(f(\v), f(\w))$ $\forall \v, \w \in V$, e dunque che $f$ è un'isometria, come desiderato dalla tesi. \end{proof} \begin{proposition} $(V, \varphi)$ e $(V', \varphi')$ spazi vettoriali su $\RR$ sono isometrici $\iff$ $\varphi$ e $\varphi'$ hanno la stessa segnatura. \end{proposition} \begin{proof}\nl\nl \rightproof Per la precedente proposizione, esistono due basi $\basis$ e $\basis'$, una di $V$ e una di $V'$, tali che $M_\basis(\varphi) \cong M_{\basis'}(\varphi)$. Allora queste due matrici condividono la stessa segnatura, e così quindi anche $\varphi$ e $\varphi'$. \\ \leftproof Se $\varphi$ e $\varphi'$ hanno la stessa segnatura, esistono due basi $\basis = \{ \vv 1, \ldots, \vv n \}$ e $\basis' = \{ \ww 1, \ldots, \ww n \}$, una di $V$ e una di $V'$, tali che $M = M_\basis(\varphi) = M_{\basis'}(\varphi')$ e che $M$ è una matrice di Sylvester. Allora si costruisce $f : V \to V'$ tale che $f(\vv i) = \ww i$. Esso è un isomorfismo, e per costruzione $\varphi(\vv i, \vv j) = \varphi'(\ww i, \ww j) = \varphi'(f(\vv i), f(\vv j))$ $\forall 1 \leq i, j \leq n$, da cui si conclude che $\varphi(\v, \w) = \varphi'(f(\v), f(\w))$ $\forall \v$, $\w \in V$, e quindi che $V$ e $V'$ sono isometrici. \end{proof} \begin{definition}[sottospazio isotropo] Sia $W$ un sottospazio di $V$. Allora $W$ si dice \textbf{sottospazio isotropo} di $V$ se $\restr{\varphi}{W} = 0$. \end{definition} \begin{remark}\nl \li $V^\perp$ è un sottospazio isotropo di $V$. \\ \li $\vec{v}$ è un vettore isotropo $\iff$ $W = \Span(\vec v)$ è un sottospazio isotropo di $V$. \\ \li $W \subseteq V$ è isotropo $\iff$ $W \subseteq W^\perp$. \end{remark} \begin{proposition} Sia $\varphi$ non degenere. Se $W$ è un sottospazio isotropo di $V$, allora $\dim W \leq \frac{1}{2} \dim V$. \end{proposition} \begin{proof} Poiché $W$ è un sottospazio isotropo di $V$, $W \subseteq W^\perp \implies \dim W \leq \dim W^\perp$. Allora, poiché $\varphi$ è non degenere, $\dim W + \dim W^\perp = \dim V$, $\dim W \leq \dim V - \dim W$, da cui $\dim W \leq \frac{1}{2} \dim V$. \end{proof} \begin{definition}[indice di Witt] Si definisce \textbf{indice di Witt} $W(\varphi)$ di $(V, \varphi)$ come la massima dimensione di un sottospazio isotropo. \end{definition} \begin{remark}\nl \li Se $\varphi > 0$ o $\varphi < 0$, $W(\varphi) = 0$. \end{remark} \begin{proposition} Sia $\KK = \RR$. Sia $\varphi$ non degenere e sia $\sigma(\varphi) = (\iota_+(\varphi), \iota_-(\varphi), 0)$. Allora $W(\varphi) = \min\{\iota_+(\varphi), \iota_-(\varphi)\}$. \end{proposition} \begin{proof} Senza perdità di generalità si assuma $\iota_-(\varphi) \leq \iota_+(\varphi)$ (il caso $\iota_-(\varphi) > \iota_+(\varphi)$ è analogo). Sia $W$ un sottospazio con $\dim W > \iota_-(\varphi)$. Sia $W^+$ un sottospazio con $\dim W^+ = \iota_+(\varphi)$ e $\restr{\varphi}{W^+} > 0$. Allora, per la formula di Grassmann, $\dim W + \dim W^+ > n \implies \dim W + \dim W^+ > \dim W + \dim W^+ - \dim (W \cap W^+) \implies \dim (W \cap W^+) > 0$. Quindi $\exists \w \in W$, $\w \neq \vec 0$ tale che $\varphi(\w, \w) > 0$, da cui si ricava che $W$ non è isotropo. Pertanto $W(\varphi) \leq \iota_-(\varphi)$. \\ Sia $a := \iota_+(\varphi)$ e sia $b := \iota_-(\varphi)$. Sia ora $\basis = \{ \vv 1, \ldots, \vv a, \ww 1, \ldots, \ww b \}$ una base tale per cui $M_\basis(\varphi)$ è la matrice di Sylvester per $\varphi$. Siano $\vv 1$, ..., $\vv a$ tali che $\varphi(\vv i, \vv i) = 1$ con $1 \leq i \leq a$. Analogamente siano $\ww 1$, ..., $\ww b$ tali che $\varphi(\ww i, \ww i) = -1$ con $1 \leq i \leq b$. Detta allora $\basis' = \{ \vv 1 ' := \vv 1 + \ww 1, \ldots, \vv b ' := \vv b + \ww b \}$, sia $W = \Span(\basis')$. \\ Si osserva che $\basis'$ è linearmente indipendente, e dunque che $\dim W = \iota_-$. Inoltre $\varphi(\vv i ', \vv j ') = \varphi(\vv i + \ww i, \vv j + \ww j)$. Se $i \neq j$, allora $\varphi(\vv i ', \vv j ') = 0$, dal momento che i vettori di $\basis$ sono a due a due ortogonali tra loro. Se invece $i = j$, allora $\varphi(\vv i ', \vv j ') = \varphi(\vv i, \vv i) + \varphi(\ww i, \ww i) = 1-1=0$. Quindi $M_{\basis'}(\restr{\varphi}{W}) = 0$, da cui si conclude che $\restr{\varphi}{W} = 0$. Pertanto $W(\varphi) \geq i_-(\varphi)$, e quindi $W(\varphi) = i_-(\varphi)$, da cui la tesi. \end{proof} \end{document}